Interdisciplinary Research

This topic has expert replies
User avatar
Legendary Member
Posts: 1132
Joined: Mon Jul 20, 2009 3:38 am
Location: India
Thanked: 64 times
Followed by:6 members
GMAT Score:760

Interdisciplinary Research

by harsh.champ » Fri Feb 19, 2010 2:07 pm
Most scientists agree that new lines of interdisciplinary research are the need of the hour. Even government committees on science have stressed the need for more interdisciplinary projects. Yet, of ten proposals for new interdisciplinary projects last year, only one was successfully funded. Some have suggested that this means that as yet researchers are not coming up with sufficiently persuasive projects, or that their proposals are not of high enough quality, or even that the reputations of these researchers is not high enough. However, the real reason probably lies in the way funding is organized. Funding is still allocated according to the old categories and there are no funds specifically for research that overlaps different subject areas.

The two parts in bold-face are related to each other in which of the following ways?

A. The first is a finding that the author finds unacceptable; the second is the author's own position
B. The first is a finding that the author attempts to account for; the second is a finding that contradicts the author's main conclusion.
C. The first is a fact that the author attempts to account for. The second is data that explicitly supports the author's main conclusion.
D. The first is a position that the author opposes; the second is the author's main position.
E. The first is a situation that the author finds paradoxical; the second is an assumption that the author uses to reinforce the paradox.
It takes time and effort to explain, so if my comment helped you please press Thanks button :)



Just because something is hard doesn't mean you shouldn't try,it means you should just try harder.

"Keep Walking" - Johnny Walker :P

User avatar
Legendary Member
Posts: 1560
Joined: Tue Nov 17, 2009 2:38 am
Thanked: 137 times
Followed by:5 members

by thephoenix » Sat Feb 20, 2010 8:49 pm
harsh.champ wrote:Most scientists agree that new lines of interdisciplinary research are the need of the hour. Even government committees on science have stressed the need for more interdisciplinary projects. Yet, of ten proposals for new interdisciplinary projects last year, only one was successfully funded. Some have suggested that this means that as yet researchers are not coming up with sufficiently persuasive projects, or that their proposals are not of high enough quality, or even that the reputations of these researchers is not high enough. However, the real reason probably lies in the way funding is organized. Funding is still allocated according to the old categories and there are no funds specifically for research that overlaps different subject areas.

The two parts in bold-face are related to each other in which of the following ways?

A. The first is a finding that the author finds unacceptable; the second is the author's own position
B. The first is a finding that the author attempts to account for; the second is a finding that contradicts the author's main conclusion.
C. The first is a fact that the author attempts to account for. The second is data that explicitly supports the author's main conclusion.
D. The first is a position that the author opposes; the second is the author's main position.
E. The first is a situation that the author finds paradoxical; the second is an assumption that the author uses to reinforce the paradox.
IMO E

yet is a counter premise by contradicting the author creates a paradox , and E states that

Master | Next Rank: 500 Posts
Posts: 113
Joined: Thu Feb 26, 2009 8:13 am
Location: New Jersey
GMAT Score:650

by KICKGMATASS123 » Mon Feb 22, 2010 2:22 am
harsh.champ wrote:Most scientists agree that new lines of interdisciplinary research are the need of the hour. Even government committees on science have stressed the need for more interdisciplinary projects. Yet, of ten proposals for new interdisciplinary projects last year, only one was successfully funded. Some have suggested that this means that as yet researchers are not coming up with sufficiently persuasive projects, or that their proposals are not of high enough quality, or even that the reputations of these researchers is not high enough. However, the real reason probably lies in the way funding is organized. Funding is still allocated according to the old categories and there are no funds specifically for research that overlaps different subject areas.

The two parts in bold-face are related to each other in which of the following ways?

A. The first is a finding that the author finds unacceptable; the second is the author's own position
B. The first is a finding that the author attempts to account for; the second is a finding that contradicts the author's main conclusion.
C. The first is a fact that the author attempts to account for. The second is data that explicitly supports the author's main conclusion.
D. The first is a position that the author opposes; the second is the author's main position.
E. The first is a situation that the author finds paradoxical; the second is an assumption that the author uses to reinforce the paradox.
IMO C

Master | Next Rank: 500 Posts
Posts: 239
Joined: Wed Feb 11, 2009 2:50 am

by delhiboy1979 » Mon Feb 22, 2010 3:41 am
Agree with C. Looks the bes of the options

User avatar
Newbie | Next Rank: 10 Posts
Posts: 3
Joined: Tue Mar 03, 2009 8:16 pm

by abhi_nakshatrala » Tue Feb 23, 2010 3:14 am
FACTS NOTICED
of ten proposals for new interdisciplinary projects last year, only one was successfully funded

CONCLUSION
However, the real reason probably lies in the way funding is organized.

SUPPORT FOR THE ABOVE CONCLUSION
Funding is still allocated according to the old categories and there are no funds specifically for research that overlaps different subject areas.


IMO C. Can we have the OA please?

User avatar
Master | Next Rank: 500 Posts
Posts: 176
Joined: Sun Feb 21, 2010 6:00 am
Thanked: 2 times
GMAT Score:710

by Pdgmat2010 » Tue Feb 23, 2010 3:42 am
IMO E

User avatar
Legendary Member
Posts: 526
Joined: Sat Feb 21, 2009 11:47 pm
Location: India
Thanked: 68 times
GMAT Score:680

by harshavardhanc » Tue Feb 23, 2010 6:45 am
abhi_nakshatrala wrote:FACTS NOTICED
of ten proposals for new interdisciplinary projects last year, only one was successfully funded

CONCLUSION
However, the real reason probably lies in the way funding is organized.

SUPPORT FOR THE ABOVE CONCLUSION
Funding is still allocated according to the old categories and there are no funds specifically for research that overlaps different subject areas.


IMO C. Can we have the OA please?
the part that has been highlighted is not the conclusion. In fact, author does not conclude anything in this argument.

He starts with "Most scientists believe...", which is a fact. After this, he presents several other facts and provides possible explanation for one of them. This explanation is, therefore, not the conclusion.

C is a 50-50 split in which the first part is correct, whereas, the second is not.

look at this part of stimulus :

" Even government committees on science have stressed the need for more interdisciplinary projects. Yet, of ten proposals for new interdisciplinary projects last year, only one was successfully funded "


meaning : despite committees' stressing, only 1/10th of projects are funded. Its a paradox.

and then the author tries to give reason for it, but uses possibly, which means he's not sure. May be/may be not his reason is correct.

E paraphrases this situation.

IMO E
Regards,
Harsha

Senior | Next Rank: 100 Posts
Posts: 99
Joined: Sun Jul 26, 2009 7:56 pm
Thanked: 5 times
GMAT Score:720

by singhag » Wed Feb 24, 2010 8:02 am
IMO C ...second part clearly support the conclusion

It should not be E because second part does not provide assumption to reinforce assumption whereas it resolve the paradox..if first one is treated as paradox.
Last edited by singhag on Wed Feb 24, 2010 10:20 am, edited 1 time in total.

GMAT Instructor
Posts: 1302
Joined: Mon Oct 19, 2009 2:13 pm
Location: Toronto
Thanked: 539 times
Followed by:164 members
GMAT Score:800

by Testluv » Wed Feb 24, 2010 10:09 am
The correct answer is choice C.

The first bold statement presents facts for which the author is offering an explanation. His explanation comes in the second-to-last sentence of the passage. The last sentence (the second bold statement) is support for that explanation.

As an aside, choice E is necessarily incorrect because it says that the statement is as an "assumption"; however, assumptions are always UNSTATED.
Kaplan Teacher in Toronto

User avatar
Legendary Member
Posts: 526
Joined: Sat Feb 21, 2009 11:47 pm
Location: India
Thanked: 68 times
GMAT Score:680

by harshavardhanc » Wed Feb 24, 2010 10:43 am
Testluv wrote:The correct answer is choice C.

The first bold statement presents facts for which the author is offering an explanation. His explanation comes in the second-to-last sentence of the passage. The last sentence (the second bold statement) is support for that explanation.

As an aside, choice E is necessarily incorrect because it says that the statement is as an "assumption"; however, assumptions are always UNSTATED.
so you mean to say that explanations are conclusions ? I don't think so. In fact, author is not concluding anything in this argument. He is just presenting facts and is giving possible cause for one of them.

agree that E's second half limps....but C suffers from the same disease.
Regards,
Harsha

GMAT Instructor
Posts: 1302
Joined: Mon Oct 19, 2009 2:13 pm
Location: Toronto
Thanked: 539 times
Followed by:164 members
GMAT Score:800

by Testluv » Wed Feb 24, 2010 7:09 pm
so you mean to say that explanations are conclusions ? I don't think so. In fact, author is not concluding anything in this argument. He is just presenting facts and is giving possible cause for one of them.

agree that E's second half limps....but C suffers from the same disease.
Yes, explanations (or causes) are indeed conclusions because they are assertions of fact. In fact, the most common form of argument that shows up in stn/wkn questions is that of "explaining the phenomenon" or "asserting a cause". (Also, if there weren't a conclusion, then it wouldn't even be an argument! To be an argument, you need to argue towards something; and the thing you are arguing towards is your conclusion--your point.)

Here, the first bold statement illustrates a phenomenon for which the author is asserting an explantion. (phenomenon = set of facts that stand in need of explanation). The fact that of ten proposals only one was successful requires an explanation, given that scientists agree that there is a need for interdisciplinary research.

The next sentence reads: "Some have suggested that this means that as yet researchers are not coming up with sufficiently persuasive projects, or that their proposals are not of high enough quality, or even that the reputations of these researchers is not high enough."

The "some have suggested" tells us that this is other people's explanation. The sentence that follows is the author's own explanation for that phenomenon. The final sentence (the second bold) is support for that explanation.

Therefore, choice c is demonstrably and categorically correct, and it does not suffer from the "same disease" as choice E.

Choice E is wrong because it says "assumption". All assumptions are UNSTATED. So, how can the role of a STATEMENT ever be that of assumption?

In boldface questions, you can automatically eliminate choices that say "assumption"; "presupposition", etc.

This:

"and then the author tries to give reason for it, but uses possibly, which means he's not sure. May be/may be not his reason is correct." (your comment)

does not change it from being a conclusion. Instead, it just relates to his degree of confidence. There are three degrees of confidence:

certain (100 percent);
probable (greater than 50 percent);
and possible (greater than 0 percent).

So these are all conclsions but with varying degrees of confidence:

"cholesterol does clog your arteries"
"cholesterol likely clogs your arteries"
"cholesterol may clog your areteries".
Kaplan Teacher in Toronto

Legendary Member
Posts: 610
Joined: Fri Jan 15, 2010 12:33 am
Thanked: 47 times
Followed by:2 members

by kstv » Wed Feb 24, 2010 8:27 pm
I went straight for E, basing it on Yet, ....but the word assumption weakens it.
It must be C.

GMAT Instructor
Posts: 1302
Joined: Mon Oct 19, 2009 2:13 pm
Location: Toronto
Thanked: 539 times
Followed by:164 members
GMAT Score:800

by Testluv » Wed Feb 24, 2010 8:39 pm
kstv wrote:I went straight for E, basing it on Yet, ....but the word assumption weakens it.
It must be C.
The other problem with choice E is that it says that he is trying to "reinforce the paradox". That just doesn't make any sense. He is trying to resolve or reconcile the paradox--not reinforce it.
Kaplan Teacher in Toronto

Master | Next Rank: 500 Posts
Posts: 303
Joined: Sat Aug 22, 2015 10:23 am

by joseph32 » Mon May 16, 2016 12:05 am
I like the explanation on E

Master | Next Rank: 500 Posts
Posts: 303
Joined: Sat Aug 22, 2015 10:23 am

by joseph32 » Mon May 16, 2016 12:06 am
I would go for E as well. It seems the best and safest among the rest.